Está en la página 1de 51

GMAT Statistics and Sets

Je Sackmann / GMAT HACKS February 2008

Contents
1 Introduction 2 Di culty Levels 3 Problem Solving 4 Data Su ciency 5 Answer Key 6 Explanations 2 3 4 20 26 29

1. INTRODUCTION

Je Sackmann GMAT Tutor jesackmann@gmail.com www.gmathacks.com

Introduction

This document contains nothing but GMAT questions on the topics of Statistics and Sets 100 of them, to be exact. The GMAT loves these types of questions, and has steadily increased the number of them on the test. It s almost guaranteed that, on test day, you ll see items that focus on subjects such as mean, median, standard deviation, and overlapping sets. As in all of my GMAT preparation resources, you ll nd these questions indexed by di culty. That doesn t mean you should skip straight to the hardest questions, or even that you should start with the easier ones. On the GMAT itself, questions won t come labeled with their di culty level, and despite the intent of the adaptive algorithm, they won t be precisely consistent in terms of di culty either. Each question presents its own unique challenges, and the sooner you get accustomed to changing gears with every single question, the more time you ll have to prepare for that particular challenge of the exam. For additional practice, I have produced several other resources that may help you. You ll nd the most Statistics and Sets-related questions in "Word Problems: Challenge" and "Word Problems: Fundamentals." There are also plenty in the "Arithmetic: Challenge" and "Arithmetic: Fundamentals" sets, especially those dealing with averages and overlapping sets. Eventually, you ll start seeing questions that look familiar. That s a good thing: there are only so many ways the GMAT can test these concepts, and if you ve done a few hundred Rates, Ratios, and Percents questions, you ve seen just about every permutation they can throw your way. Also, The GMAT Math Bible has several chapters (along with focused practice) on these topics, including individual chapters on averages, weighted averages, statistics such as mean, median, mode, range, and standard deviation, and overlapping sets. If you nd you are struggling with the mechanics of these problems, your time is probably better spent with the GMAT Math Bible than in doing dozens and dozens of practice problems, hoping to pick up those skills along the way. As far as strategy is concerned, there are dozens of articles at GMAT HACKS to help you with your strategic approach to Arithmetic questions. Most importantly, you should make sure you understand every practice problem you do. It doesn t matter if you get it right the rst time what matters is whether you ll get it right the next time you see it, because the next time you see it could be on the GMAT. With that in mind, carefully analyze the explanations. Redo questions that took you too long the rst time around. Review questions over multiple sessions, rather than cramming for eight hours straight each Saturday. These basic study skills may not feel like the key to GMAT preparation, but they are the dierence between those people who reach their score goals and those who never do. Enough talking; there are 100 Statistics and Sets questions waiting inside. Get to work! 2 Copyright 2008 Je Sackmann jesackmann@gmail.com - www.gmathacks.com

2. DIFFICULTY LEVELS

Je Sackmann GMAT Tutor jesackmann@gmail.com www.gmathacks.com

Di culty Levels

In general, the level 5 questions in this guide are 560- to 620-level questions. The level 6 questions represent a broad range of di culty from about 620 to 720, while the level 7 questions are higher still. Easy (4) PS 5, 12, 14, 35, 37, 45, 52, 54 DS 61, 66, 68, 71, 74, 82, 83, 84, 88, 89, 90 Moderate (5) PS 1, 4, 6, 7, 8, 9, 11, 16, 18, 20, 21, 24, 27, 28, 29, 30, 33, 34, 36, 39, 40, 42, 43, 46, 48, 49, 51, 53, 55, 57, 60 DS 63, 65, 69, 72, 73, 77, 79, 80, 85, 86, 87, 92, 94, 96, 97, 98, 100 Di cult (6) PS 2, 10, 13, 15, 17, 19, 22, 23, 26, 31, 32, 38, 44, 47, 50, 56, 58, 59 DS 62, 64, 70, 75, 76, 81, 91, 93, 95, 99 Very Di cult (7) PS 3, 25, 41 DS 67, 78

3 Copyright 2008 Je Sackmann jesackmann@gmail.com - www.gmathacks.com

3. PROBLEM SOLVING

Je Sackmann GMAT Tutor jesackmann@gmail.com www.gmathacks.com

Problem Solving

Note: this guide contains both an answer key (so you can quickly check your answers) and full explanations. 1. A law o ce employs seven secretaries. It pays annual salaries of $17,000 to 2 of the secretaries, $19,000 to 2 of the secretaries, and $23,000 to the remaining 3 secretaries. The average (arithmetic mean) annual salary of these employees is closest to which of the following? (A) $19,000 (B) $19,700 (C) $20,000 (D) $20,100 (E) $21,000 A freelance writer found that last year his average (arithmetic mean) revenue from January to October $4,800. In November and December, his monthly revenues were 2 and 3 times, respectively, the average for the other 10 months. What was the writer s average monthly revenue last year? (A) $5,400 (B) $6,000 (C) $6,400 (D) $6,800 (E) $7,200 If x is to be chosen at random from the set {1, 2, 3, 4} and y is to be chosen at random from the set {4, 5, 6, 7}, what is the probability that xy will be odd? 1 (A) 8 1 (B) 4 1 (C) 2 3 (D) 4 7 (E) 8

2.

3.

4 Copyright 2008 Je Sackmann jesackmann@gmail.com - www.gmathacks.com

3. PROBLEM SOLVING

Je Sackmann GMAT Tutor jesackmann@gmail.com www.gmathacks.com

4.

I. 80, 81, 82, 83, 84 II. 82, 82, 82, 82, 82 III. 69, 82, 82, 82, 96 The data sets I, II, and III above are ordered from greatest standard deviation to least standard deviation in which of the following? (A) I, II, III (B) I, III, II (C) II, III, I (D) III, I, II (E) III, II, I The amounts of rainfall in inches recorded in six dierent cities in a certain state last month were 12.5", 11.5", 10.8", 17", 18.2", and 15". What is the median of those amounts? (A) 14" (B) 13.75" (C) 13.2" (D) 13" (E) 12.5" The arithmetic mean and standard deviation of a certain normal distribution are 11.5 and 2.0, respectively. What value is exactly 2 standard deviations less than the mean? (A) 7.5 (B) 8.0 (C) 8.5 (D) 9.0 (E) 9.5
3 1 1 If S = { 1 3 , 5 , 0, 2 , 1, 6 }, what is the positive dierence between the median of the numbers in S and the mean of the numbers in S? 1 (A) 12 1 (B) 15 1 (C) 30 1 (D) 60 1 (E) 120

5.

6.

7.

5 Copyright 2008 Je Sackmann jesackmann@gmail.com - www.gmathacks.com

3. PROBLEM SOLVING

Je Sackmann GMAT Tutor jesackmann@gmail.com www.gmathacks.com

8.

1 1 , 0, 1 { 7, 6, 2, 4 4 , 2 , 1, 2, 4, 6, 7} A number is to be selected at random from the set above. What is the probability that the number selected will be a solution of the equation (y + 4)(y 7)(4y + 1) = 0? 1 (A) 12 1 (B) 6 1 (C) 4 1 (D) 3 1 (E) 2

9.

If a certain sample of data has a mean of 30.0 and a standard deviation of 2.5, all of the following values are more than 2.5 standard deviations from the mean EXCEPT (A) 22.0 (B) 22.5 (C) 23.5 (D) 36.0 (E) 36.5 Of the 300 students at a certain university in State S, 210 were born in State S and 250 attended high school in State S. If at least 40 of the students were neither born nor attended high school in State S, then the number of students who were both born and attended high school in State S could be any number from (A) 40 to 80 (B) 80 to 110 (C) 90 to 210 (D) 200 to 210 (E) 200 to 250 Jaclyn purchased 3 picture frames with an average (arithmetic mean) price of $18. If, after Jaclyn purchases another picture frame, the average price of the 4 picture frames is $20, what is the price of the fourth picture frame? (A) $19 (B) $20 (C) $22 (D) $26 (E) $30

10.

11.

6 Copyright 2008 Je Sackmann jesackmann@gmail.com - www.gmathacks.com

3. PROBLEM SOLVING

Je Sackmann GMAT Tutor jesackmann@gmail.com www.gmathacks.com

12.

$15, $18, $20, $24, $25, $30, $50, $50, $64, $80 The total amounts paid by a series of customers at a certain store are shown above. How many amounts were greater than the median amount but less than the mean amount? (A) None (B) One (C) Two (D) Three (E) Four In a certain neighborhood, ve houses are listed for sale at the following prices: $92,000, $98,000, $112,000, $115,000, and $128,000. If the price of the most expensive house is increased $6,000 and the price of the least expensive home is decreased by the same amount, which of the following best describes the change in the mean and the median of the house prices? (A) The mean and the median will remain unchanged. (B) The mean will remain unchanged but the median will increase. (C) The mean will increase but the median will remain unchanged. (D) The mean and the median will increase by the same amount. (E) The mean and the median will increase by dierent amounts. The average (arithmetic mean) of 40, 60, and 80 is 5 more than the average of 30, 50, and (A) 45 (B) 55 (C) 65 (D) 75 (E) 85

13.

14.

7 Copyright 2008 Je Sackmann jesackmann@gmail.com - www.gmathacks.com

3. PROBLEM SOLVING

Je Sackmann GMAT Tutor jesackmann@gmail.com www.gmathacks.com

15.

A researcher tracked the daily changes in price of all the stocks on a certain exchange for a period of one month, and found that the mean daily price change of the stocks on the exchange was $0.16, and the standard deviation of the price changes was $0.04. The largest price change the researcher observed took place on the 21st of last month, and was between 8 and 9 standard deviations above the mean. Which of the following could have been the dollar value of the largest price change observed last month? (A) $0.25 (B) $0.28 (C) $0.35 (D) $0.42 (E) $0.49 A certain typing service charges $50 to type a document of up to 20 pages and $15 for each additional 10 pages, or portion thereof. If Paul employed the service to type a 100-page document, what would be the average (arithmetic mean) charge per page? (A) $0.65 (B) $1.70 (C) $1.75 (D) $1.90 (E) $2.00 A set of 25 dierent integers has a median of 50 and a range of 50. What is the greatest possible integer that could be in this set? (A) 62 (B) 68 (C) 75 (D) 88 (E) 100 Of the 11 students who took the nal exam in a biology class, 3 scored a 72, 2 scored a 93, 3 scored a 76, 1 scored a 78, and 2 scored an 89. What was the median score on the nal exam? (A) 72 (B) 76 (C) 78 (D) 89 (E) 93

16.

17.

18.

8 Copyright 2008 Je Sackmann jesackmann@gmail.com - www.gmathacks.com

3. PROBLEM SOLVING

Je Sackmann GMAT Tutor jesackmann@gmail.com www.gmathacks.com

19.

If m is the standard deviation of x, y , and z , what is the standard deviation of x 5, y 5, and z 5? 1 (A) 3m (B) m (C) 5m (D) m 5 (E) m 15 List M consists of 15 consecutive integers. If 4 is the greatest integer in list M, what is the range of the negative integers in list M? (A) 8 (B) 9 (C) 10 (D) 14 (E) 15 A certain doughnut shop has six employees. It pays annual salaries of $15,000 to each of 3 employees, $17,000 to 1 employee, and $18,000 to each of the remaining 2 employees. The average (arithmetic mean) annual salary of these employees is closest to which of the following? (A) $15,800 (B) $16,300 (C) $16,600 (D) $16,800 (E) $17,000 A list of measurements in increasing order is 2, 4, 5, 7, 15, and x. If the median of these measurements is 2 3 their arithmetic mean, what is the value of x ? (A) 16 (B) 18 (C) 19 (D) 21 (E) 23

20.

21.

22.

9 Copyright 2008 Je Sackmann jesackmann@gmail.com - www.gmathacks.com

3. PROBLEM SOLVING

Je Sackmann GMAT Tutor jesackmann@gmail.com www.gmathacks.com

23.

In a certain graduating class, a grade-point average of 2.8 was 1.5 standard deviations below the mean, and a grade-point average of 3.6 was 2.5 standard deviations above the mean. What the mean grade-point average of the graduating class? (A) 3.4 (B) 3.3 (C) 3.2 (D) 3.1 (E) 3.0 The average (arithmetic mean) of the 4 numbers k , 2k 3k + 2, and 6k 1 is 17, what is the value of k ? (A) 4 (B) 5 (C) 6 (D) 8 (E) 9 5,

24.

25.

A certain list of 200 test scores has an average (arithmetic mean) of 85 and a standard deviation of d, where d is positive. Which of the following two test scores, when added to the list, must result in a list of 202 test scores with a standard deviation less than d ? (A) 80 and 80 (B) 80 and 85 (C) 80 and 90 (D) 85 and 85 (E) 85 and 90 If 0 < z < 1, what is the median of the values z , z and z 3 ? (A) z (B) z p 1 (C) z (D) z2 (E) z3 1, p z, z2,

26.

10 Copyright 2008 Je Sackmann jesackmann@gmail.com - www.gmathacks.com

3. PROBLEM SOLVING

Je Sackmann GMAT Tutor jesackmann@gmail.com www.gmathacks.com

27.

Three adults have an average (arithmetic mean) weight of 160 pounds and a median weight of 180 pounds. What is the maximum possible weight, in pounds, of the lightest of the three adults? (A) 109 (B) 110 (C) 119 (D) 120 (E) 140 An investor purchased 30 shares of a certain stock at a price of $26.50 per share. Later this investor purchased 20 more shares at a price of $25.50 per share. What was the average (arithmetic mean) price per share that this investor paid for the 50 shares? (A) $25.90 (B) $26.00 (C) $26.10 (D) $26.25 (E) $26.30 In a certain normal distribution, the arithmetic mean is 9.5 and the standard deviation is 1.75. What value is exactly 2 standard deviations from the mean? (A) 6 (B) 6.75 (C) 7 (D) 12.25 (E) 14 Which of the following is equal to the average (arithmetic mean) of (x + 2)2 and (x + 2)(x 2) ? (A) x2 (B) x2 + 2 (C) x2 + 4 (D) x2 + 2x (E) x2 + 4x

28.

29.

30.

11 Copyright 2008 Je Sackmann jesackmann@gmail.com - www.gmathacks.com

3. PROBLEM SOLVING

Je Sackmann GMAT Tutor jesackmann@gmail.com www.gmathacks.com

31.

W represents the sum of the weights of s steaks in pounds. Which of the following represents the average (arithmetic mean) of the s weights in ounces? (1 pound = 16 ounces) (A) 16W s 16W (B) s (C) (D) (E)
Ws 16 W 16s s 16W

32.

Forty percent of the employees at a brokerage rm have passed a certain licensing exam. Among the employees who have not passed the exam, 32 are exempt from the licensing requirement and 16 are not exempt. How many employees does the brokerage rm have? (A) 60 (B) 80 (C) 96 (D) 108 (E) 120 The maximum temperatures, in degrees Celcius, recorded in a city on 5 consecutive days were 32, y + 2, y + 5, 30, and y . If the average (arithmetic mean) of these temperatures was 24 degrees Celcius, what is the value of y ? (A) 17 (B) 19 (C) 21 (D) 22 (E) 23 Three students reported that the amount of time they spent preparing for a certain exam was between 0 and 10 hours, inclusive. If the average (arithmetic mean) number of hours the students reported that they spent preparing was 7.5 hours, what was the least possible number of hours that one of the students spent preparing, in hours? (A) 2 (B) 2.5 (C) 4 (D) 4.5 (E) 6

33.

34.

12 Copyright 2008 Je Sackmann jesackmann@gmail.com - www.gmathacks.com

3. PROBLEM SOLVING

Je Sackmann GMAT Tutor jesackmann@gmail.com www.gmathacks.com

35.

The one-day price changes in dollars of several stocks on a certain exchange were $0:25, $0:05, $0:10, $0:05, $0:25, and $0:60. What is the median of these price changes? (A) $0:10 (B) $0:05 (C) $0:00 (D) $0:05 (E) $0:25 Of the 120 passengers on an airplane, 50 had 2 pieces of luggage each, 40 had 1 piece of luggage each, 10 had 3 pieces of luggage each, 10 had 4 pieces of luggage each, and the remainder had no luggage. What was the average (arithmetic mean) number of pieces of luggage per passenger on the airplane? (A) 1 (B) 1.25 (C) 1.5 (D) 1.75 (E) 2 69 78 78 79 80 82 84 90 The mean and the approximate standard deviation of the 8 numbers shown are 80 and 6, respectively. What percent of the 8 numbers are within 1 standard deviation of the mean? (A) 87.5% (B) 82.5% (C) 80% (D) 75% (E) 62.5% Route A: 35 Route B: 25 Route C: 20 The table above shows the number of pilots who y three routes for an airline. Although none of the pilots ies all three routes, 7 pilots y both A and B, 4 pilots y both A and C, and 3 pilots y both B and C. How many dierent pilots y these three routes for the airline? (A) 64 (B) 66 (C) 67 (D) 73 (E) 75

36.

37.

38.

13 Copyright 2008 Je Sackmann jesackmann@gmail.com - www.gmathacks.com

3. PROBLEM SOLVING

Je Sackmann GMAT Tutor jesackmann@gmail.com www.gmathacks.com

39.

A realtor sold 15 of the new homes in a certain neighborhood in April, and sold the remaining 10 in May. In April, the range of the selling prices was $22,000 and the lowest selling price was $184,000. In May, the range of the selling prices was $25,000 and the lowest selling price was $192,000. What was the range of the selling prices of the 25 homes that the realtor sold in April and May? (A) $23,500 (B) $27,500 (C) $28,000 (D) $30,000 (E) $33,000 A rm purchased two machines whose prices were $5,000 and $9,000, respectively. The rm is to purchase two more machines from a list of machines whose prices range from $5,000 to $9,000, inclusive. The greatest possible average (arithmetic mean) price of the 4 machines is how much greater than the least possible average price of the 4 machines? (A) $2,000 (B) $2,500 (C) $3,000 (D) $3,500 (E) $4,000 If z is the standard deviation of p, q , and r, what is the standard deviation of 3p, 3q , and 3r ? (A) z (B) 3z (C) z+ 3 (D) z+ 9 (E) It cannot be determined from the information given. A retailer has a goal of selling $250,000 worth of software in 100 days. If it sold $170,000 in the rst 60 days, what is the average (arithmetic mean) number of dollars per day of software that it must sell for the last 40 days in order to achieve its goal? (A) $1,333 (B) $1,500 (C) $1,750 (D) $2,000 (E) $2,500

40.

41.

42.

14 Copyright 2008 Je Sackmann jesackmann@gmail.com - www.gmathacks.com

3. PROBLEM SOLVING

Je Sackmann GMAT Tutor jesackmann@gmail.com www.gmathacks.com

43.

35, 37.5, 40, 42.5, 45, 52.5, 52.5, 55, 60, 60 The list shown consists of the heights, in inches, of each of 10 schoolchildren. If the standard deviation of the heights is 9.2 inches, rounded to the nearest tenth of an inch, how many of the 10 heights are more than 1 standard deviation away from the mean of the 10 heights? (A) Two (B) Three (C) Four (D) Five (E) Six In a survey of 232 people, 153 own their own home, 80 have children, and 1 3 of those who own their own home have children. If a person is to be randomly selected from those surveyed, what is the probability that the person selected will have children but does not own his or her own home? 1 (A) 8 29 (B) 153 25 (C) 116 51 (D) 232 1 (E) 4 The number of credits being taken by each of eight students at a certain university are 16, 12, 11, 15, 8, 6, 14, and 15. What is the range in the number of credits being taken by the eight students? (A) 8 (B) 9 (C) 10 (D) 11 (E) 12 An investor s portfolio increased in value by $15,000 in 1992 and by $25,000 in 1993. In 1994 the portfolio decreased in value by $17,500. What was the portfolio s average (arithmetic mean) increase in value for the 3 years? (A) $5,000 (B) $7,500 (C) $10,000 (D) $12,500 (E) $15,000

44.

45.

46.

15 Copyright 2008 Je Sackmann jesackmann@gmail.com - www.gmathacks.com

3. PROBLEM SOLVING

Je Sackmann GMAT Tutor jesackmann@gmail.com www.gmathacks.com

47.

In a random sample of 20 members of a certain health club last week, 3 did not attend any classes, 8 each attended one class, 5 each attended 2 classes, and the remaining members in the sample attended at least 3 classes. If the average (arithmetic mean) number of classes attended by each member was 1.75, what is the maximum number of classes than any single member could have attended? (A) 4 (B) 5 (C) 6 (D) 8 (E) 10 Last week, Franka recorded the time that she spent each day in a morning meeting. The times, in minutes, were 52, 28, 40, 39, and 51. How many minutes greater was the average (arithmetic mean) time than the median time? (A) 1.5 (B) 2 (C) 2.5 (D) 3 (E) 3.5 Hank, Jelena, and Kristof each called customer support for a certain product. Hank called customer support 5 times and Jelena called customer support 2 times. If the 3 people called customer support an average (arithmetic mean) of 3 times per person, how many times did Kristof call customer support? (A) 1 (B) 2 (C) 3 (D) 4 (E) 5 Each of the 15 cars a certain dealer sold last week were purchased for one of three dierent prices and the average selling price of the 15 cars was $16,000. If 6 of the cars were sold for $14,000 each and 5 of the cars were sold for $16,000 each, what was the selling price of each of the remaining 4 cars? (A) $16,000 (B) $17,500 (C) $18,000 (D) $18,500 (E) $19,000 16 Copyright 2008 Je Sackmann jesackmann@gmail.com - www.gmathacks.com

48.

49.

50.

3. PROBLEM SOLVING

Je Sackmann GMAT Tutor jesackmann@gmail.com www.gmathacks.com

51.

List I: 4, 8, 10, 15 List II: k , 4, 8, 10, 15 If the median of the numbers in list I above is equal to the median of the numbers in list II above, what is the value of k ? (A) 6 (B) 8 (C) 9 (D) 10 (E) 11 Delia had 5 conference calls on Monday, 7 conference calls on Tuesday, 8 conference calls on Wednesday, 5 conference calls on Thursday, and 12 conference calls on Friday. What was the average (arithmetic mean) number of conference calls that Delia made for those 5 days? (A) 8.0 (B) 7.8 (C) 7.6 (D) 7.4 (E) 7.2 A group of 20 people each received tax refunds, and that the average (arithmetic mean) tax refund was $4,000. If the average tax refund of 16 of the people in the group was $3,000, what was the average tax refund for the other four people in the group? (A) $4,000 (B) $5,000 (C) $6,000 (D) $7,000 (E) $8,000 The average (arithmetic mean) of 10, 15, and 20 equals the average of 12, 18, and (A) 6 (B) 10 (C) 12 (D) 15 (E) 24

52.

53.

54.

17 Copyright 2008 Je Sackmann jesackmann@gmail.com - www.gmathacks.com

3. PROBLEM SOLVING

Je Sackmann GMAT Tutor jesackmann@gmail.com www.gmathacks.com

55.

Of the 18 entrees oered on a certain menu, 10 contain meat, 12 contain cheese, and 3 contain neither meat nor cheese. How many of the entrees contain both meat and cheese? (A) 6 (B) 7 (C) 8 (D) 9 (E) 10 If the average (arithmetic mean) of a and b is 60 and the average (arithmetic mean) of b and c is 70, what is the value of c a ? (A) 65 (B) 20 (C) 10 (D) 5 (E) It cannot be determined from the information given. If 65 percent of apparel retailers in City X sell Brand A clothing, 65 percent carry Brand B, and 25 percent carry neither brand, what percent carry both brands? (A) 35 (B) 45 (C) 50 (D) 55 (E) 65 For the past x days, Joan has made an average (arithmetic mean) of 15 sales per day. If Joan makes 25 sales today and raises her daily average to 16 sales per day, what is the value of x ? (A) 4 (B) 5 (C) 9 (D) 10 (E) 12 M is the set of positive even integers less than 75, and N is the set of the square roots of the integers in M . How many elements does the intersection of M and N contain? (A) None (B) Two (C) Four (D) Five (E) Six 18 Copyright 2008 Je Sackmann jesackmann@gmail.com - www.gmathacks.com

56.

57.

58.

59.

3. PROBLEM SOLVING

Je Sackmann GMAT Tutor jesackmann@gmail.com www.gmathacks.com

60.

A rope 40 feet long is cut into 5 pieces. If one of the pieces is 16 feet long, what is the average (arithmetic mean) length, in feet, of the remaining pieces? (A) 3.2 (B) 4 (C) 4.8 (D) 6 (E) 8

19 Copyright 2008 Je Sackmann jesackmann@gmail.com - www.gmathacks.com

4. DATA SUFFICIENCY

Je Sackmann GMAT Tutor jesackmann@gmail.com www.gmathacks.com

Data Su ciency

For all Data Su ciency questions, the answer choices are as follows: (A) (B) (C) (D) (E) 61. Statement (1) ALONE is su cient, but statement (2) alone is not su cient. Statement (2) ALONE is su cient, but statement (1) alone is not su cient. BOTH statements TOGETHER are su cient, but NEITHER statement ALONE is su cient. EACH statement ALONE is su cient. Statements (1) and (2) TOGETHER are NOT su cient. If p is an integer, is q an integer? (1) The average (arithmetic mean) of p, q , and 8 is p. (2) The average (arithmetic mean) of p, q , and 3.5 is 3.5. For a certain set of n numbers, where n > 2, is the average (arithmetic mean) equal to the median? (1) The n numbers are positive, consecutive integers. (2) The smallest integer in the set is odd. If a, b, c, d, and e are dierent positive integers, which of the ve integers is the median? (1) a, b, c, d, and e are consecutive integers. (2) The average (arithmetic mean) of the ve integers is d. How many of the 250 attendees at a certain concert paid full price for their ticket and bought their ticket at least two weeks in advance? (1) Of the 250 attendees, 65 bought their ticket at least two weeks in advance but did not pay full price for their ticket. (2) Of the 250 attendees, 75 paid full price for their ticket but did not buy their ticket at least two weeks in advance. Of a physician s clients, 120 have health insurance or prescription drug coverage or both. If 40 of the clients do not have prescription drug coverage, how many of the clients have both health insurance and prescription drug coverage? (1) A total of 92 of the clients have health insurance. (2) Of the 120 clients, 28 do not have health insurance.

62.

63.

64.

65.

20 Copyright 2008 Je Sackmann jesackmann@gmail.com - www.gmathacks.com

4. DATA SUFFICIENCY

Je Sackmann GMAT Tutor jesackmann@gmail.com www.gmathacks.com

66.

If the average (arithmetic mean) of ve numbers is 60, how many of the numbers are equal to 60? (1) At least two of the numbers are greater than 60 (2) At least two of the numbers are less than 60. Set S has a range of p and the largest number in set S is v . Set T has a range of q and the largest number in set T and the largest number in set T is w. Is the smallest number in set S greater than the smallest number in set T? (1) p<q (2) The median of set S is less than the median of set T, and the average (arithmetic mean) of set S is greater than the mean of set T. What is the average (arithmetic mean) of x, y , and z ? x+y +z =9 (1) 2 (2)
x+y +z 3

67.

68.

=6

69.

If z is a positive integer, is z < 16 ? (1) z is less than the average (arithmetic mean) of the rst ten positive even integers. (2) z is the square of an integer. Of the 600 employees in a certain company, 180 drive to work and are more than 30 years old. How many of the 600 employees drive to work and are 30 years old or less? (1) 480 of the employees in the company are more than 30 years old. (2) 50 employees in the company are 30 years old or less and do not drive to work. If a, b, c, d, and e are dierent positive integers, which of the ve integers is the median? (1) b+c<e (2) a+d<e Is q equal to the median of the three positive integers n, p, and q ? (1) p = 4n = 3q (2) q = 12 If twelve consecutive even integers are listed from least greatest, what is the average (arithmetic mean) of the twelve integers? (1) The average of the rst eight integers is 13. (2) The average of the last eight integers is 21. 21 Copyright 2008 Je Sackmann jesackmann@gmail.com - www.gmathacks.com

70.

71.

72.

73.

4. DATA SUFFICIENCY

Je Sackmann GMAT Tutor jesackmann@gmail.com www.gmathacks.com

74.

What is the average (arithmetic mean) of a, b, and c ? (1) a+b=7 (2) b + c = 10 At least 100 employees in a certain company have management experience. If 15 percent of the employees in the company who have sales experience also have management experience, do more employees have sales experience than management experience? (1) 72 employees in the company have both sales experience and management experience. (2) 252 employees in the company have neither sales experience nor management experience. In the last two years, each of Jeremy s six children grew in height by at least one inch. If the standard deviation of their heights two years ago was 4.5 inches, what is the standard deviation of their heights? (1) In the last two years, the heights of Jeremy s six children have increased a total of 17 inches. (2) In the last two years, each child s height has increased by 5 percent. When scientists introduced a certain chemical into 30 dierent ponds, the population of bacteria in some of the ponds decreased, and the the population of sh in some of the ponds decreased. In how many of the ponds did the population of sh decrease but the population of bacteria did not decrease? (1) The population of sh decreased in 7 of the 30 ponds. (2) The population of both sh and bacteria stayed constant or increased in 18 of the 30 ponds. For the members of Team A, the range of their heights is a inches and the least height is j inches. For the members of Team B, the range of their heights is b inches and the least height is k inches. Is the greatest height of the members of Team A less than the greatest height of the members of Team B? (1) a<b (2) j< k

75.

76.

77.

78.

22 Copyright 2008 Je Sackmann jesackmann@gmail.com - www.gmathacks.com

4. DATA SUFFICIENCY

Je Sackmann GMAT Tutor jesackmann@gmail.com www.gmathacks.com

79.

If the average (arithmetic mean) of seven numbers is 72, how many of the numbers are equal to 72? (1) Of the seven numbers, six are odd integers, and the average of those odd integers is 72. (2) At least one of the numbers is equal to 72. How many of the 56 people in a group are blonde males? (1) 36 of the 56 people are blonde. (2) 32 of the 56 people are male. Of the 32 speakers managed by a certain talent agency, some specialize in marketing and some are willing to travel to City X. How many of the speakers specialize in marketing but are not willing to travel to City X? (1) All of the speakers who are willing to travel to City X specialize in marketing. (2) 5 of the speakers are not willing to travel to City and do not specialize in marketing. If v + z = 32, what is the value of v z ? (1) The average of v and z is 16. (2) The positive dierence between v and z is 16. If set S consists of the numbers w, x, y , and z , is the range of the numbers in S greater than 6 ? (1) w x=4 (2) x z=4 If x is a positive number less than 10, is z less than the average (arithmetic mean) of x and 10 ? (1) z = 6x (2) On the number line, z is closer to 10 than it is to x. What is the average (arithmetic mean) of j and k ? (1) The average (arithmetic mean) of j + 3 and k + 3 is 12. (2) The average (arithmetic mean) of j , k , and 6 is 8. Of the 200 belts at a certain store, 115 are made of leather. How many of the leather belts are designed for men? (1) 65 of the belts at the store are neither made of leather or designed for men. (2) 20 of the belts at the store that are designed for men are not made of leather.

80.

81.

82.

83.

84.

85.

86.

23 Copyright 2008 Je Sackmann jesackmann@gmail.com - www.gmathacks.com

4. DATA SUFFICIENCY

Je Sackmann GMAT Tutor jesackmann@gmail.com www.gmathacks.com

87.

Is the range of the integers 7, 4, x, 5, 6, and y greater than 9 ? (1) y > 4x (2) 0<x<y If m, n, p, q , and r are dierent positive integers, which of the ve integers is the median? (1) q+r <n (2) m<p If the average (arithmetic mean) of 4 numbers is 100, how many of the numbers are greater than 100? (1) One of the numbers is equal to 100. (2) One of the numbers is equal to 50. If John s average (arithmetic mean) score for three games of pinball was 112, what was his median score? (1) The average (arithmetic mean) of John s highest and lowest scores was 118. (2) John s highest score was 154. Are at least 80 percent of the people in Country X who are 25 years old or younger full-time students? (1) In Country X, 32 percent of the population is 25 years old or younger. (2) In Country X, of the population 25 years old or younger, 90 percent of the women and 75 percent of the men are full-time students. A certain credit rating rm assigned ratings of AAA, AA, or A to each of several countries. What percent of the rated countries have a population that is greater than 20 million people? (1) Of the countries that received a rating of AAA, 25 percent have a population that is greater than 20 million people. (2) Of the countries that received a rating of AA or A, 70 percent have a population that is 20 million people or fewer. Each of the terms in sets R, S, and T are positive integers. If 9 of the integers in R are also in S, 12 of the integers that are in S are also T, and 4 of the integers that are in R are also in T, how many unique positive integers are contained in the three sets? (1) 1 of the integers is in R, S, and T. (2) R has 20 terms, S has 24 terms, and T has 30 terms. 24 Copyright 2008 Je Sackmann jesackmann@gmail.com - www.gmathacks.com

88.

89.

90.

91.

92.

93.

4. DATA SUFFICIENCY

Je Sackmann GMAT Tutor jesackmann@gmail.com www.gmathacks.com

94.

Of the people who attended a conference, 75 percent were employed by biotechnology rms, and some of those employed by biotechnology rms were executives of those rms. What percent of the people who attended the conference were executives of biotechnology rms? (1) 60 percent of the employees of biotechnology rms who attended the conference were executives of those rms. (2) 300 people attended the conference. Set X consists of seven consecutive integers, and set Y consists of nine consecutive integers. Is the median of the numbers in set X equal to the median of the numbers in set Y ? (1) The sum of the numbers in set X is equal to the sum of the numbers in set Y. (2) The median of the numbers in set Y is 0. If set S consists of the numbers 3, -4, -9, 0, and m, is 6 < m < 2? (1) The median of the numbers in set S is greater than -4. (2) The median of the numbers in set S is negative. Of the six hundred children in a certain county, how many have been immunized against neither disease D or disease E? (1) Of the children in the county, 580 have been immunized against disease E. (2) Of the children in the county, 570 have been immunized against both disease D and disease E. If k is a positive odd integer, what is the average (arithmetic mean) of a certain set of k integers? (1) The median of the set of integers is 44. (2) The integers in the set are consecutive multiples of 4. Is s equal to the median of the four positive integers r, s, t, and u? (1) r is the smallest of the four integers r, s, t, and u. (2) s, t, and u are consecutive even integers. A and B are sets of positive integers. Is the least integer in A greater than the least integer in B ? (1) A is a set of 5 consecutive even integers, each less than 30. (2) B is a set of 3 consecutive odd integers, each less than 25. 25 Copyright 2008 Je Sackmann jesackmann@gmail.com - www.gmathacks.com

95.

96.

97.

98.

99.

100.

5. ANSWER KEY

Je Sackmann GMAT Tutor jesackmann@gmail.com www.gmathacks.com

Answer Key

For full explanations, see the next section. 1. 2. 3. 4. 5. 6. 7. 8. 9. 10. 11. 12. 13. 14. 15. 16. 17. 18. 19. 20. 21. 22. 23. 24. 25. 26. 27. 28. 29. 30. 31. 32. 33. 34. 35. 36. 37. 38. 39. 40. 41. D B B D B A D B D D D B A E E B D B B B B D D C D D D C A D B B A B C D D B E A B 26 Copyright 2008 Je Sackmann jesackmann@gmail.com - www.gmathacks.com

5. ANSWER KEY

Je Sackmann GMAT Tutor jesackmann@gmail.com www.gmathacks.com

42. 43. 44. 45. 46. 47. 48. 49. 50. 51. 52. 53. 54. 55. 56. 57. 58. 59. 60. 61. 62. 63. 64. 65. 66. 67. 68. 69. 70. 71. 72. 73. 74. 75. 76. 77. 78. 79. 80. 81. 82. 83. 84. 85. 86. 87.

D C A C B D B B E C D E D B B D C C D D A C E D E E D A C E A D E E B E C A E E E C B D E E 27 Copyright 2008 Je Sackmann jesackmann@gmail.com - www.gmathacks.com

5. ANSWER KEY

Je Sackmann GMAT Tutor jesackmann@gmail.com www.gmathacks.com

88. 89. 90. 91. 92. 93. 94. 95. 96. 97. 98. 99. 100.

E E A E E C A C C E C C E

28 Copyright 2008 Je Sackmann jesackmann@gmail.com - www.gmathacks.com

6. EXPLANATIONS

Je Sackmann GMAT Tutor jesackmann@gmail.com www.gmathacks.com

Explanations

For a quick-reference answer key, see the previous section. 1. D Explanation: Explanation: average problem as follows:
2(17;000)+2(19;000)+3(23;000) 7

Technically, you can set up this weighted

However, that s time-consuming and error-prone. Instead, subtract 17,000 from all the numbers (and remember to add it to your answer when you re done!). 2(0)+2(2;000)+3(6;000) 000 = 22;7 3; 000 7 Add 17,000 back to the total, and the result is a little more than 20,000: choice (D), 20,100. 2. B Explanation: To avoid doing more math than necessary, call the average monthly revenue for the rst 10 months x. (x = 4800, but this way we aren t doing a lot of unnecessary arithmetic.) The revenue in November, then, was 2x, and the revenue in December was 3x. The total revenue for the entire year was 10x + 2x + 3x = 15x. x The average revenue is 15 s the answer we re looking for, 12 = 1:25x. That once we plug x = 4800 back into the expression: 1:25x = 1:25(4800) = 6000, choice (B). 3. B Explanation: The product of two integers will be odd if and only if both of the integers are odd. 2 of the 4 possible values of x are odd, so the probability that x is odd is 1 2 . 2 of the 4 possible values of y are odd, so the probability that y is odd is also 1 2 . The probability that both x and y are odd, then, is 1 1 1 = , choice (B). 2 2 4 4. D Explanation: Standard deviation is, approximately, the average distance from the mean of each term in the set. II clearly has the smallest standard devation: if each term is the same, the standard deviation is zero. That limits our choices to (B) and (D). While III has three terms that are equal, the two outliers make the standard deviation much larger. You can probably evaluate the approximate standard deviations without doing the math, but the average dierence from the mean in III is 13+0+0+0+14 = 27 5 5 , while the average dierence from the mean in I is 2+1+0+1+2 6 = . It s not even close. Choice (D) is correct. 5 5 5. B

29 Copyright 2008 Je Sackmann jesackmann@gmail.com - www.gmathacks.com

6. EXPLANATIONS

Je Sackmann GMAT Tutor jesackmann@gmail.com www.gmathacks.com

Explanation: To nd the median, line up the measurements in ascending or descending order: 10.8, 11.5, 12.5, 15, 17, 18.2 Since there is an even number of terms in the set, the median is the mean of the two middle terms, in this case 12.5 and 15: :5 = 27 m = 12:5+15 2 2 = 13:75, choice (B). 6. A Explanation: The mean is 11.5 and 1 standard deviation is 2. Two standard deviations, then, is 2(2) = 4, so the value that is 2 standard deviations less than the mean is 11:5 4 = 7:5, choice (A). 7. D Explanation: To nd the median, arrange the terms in ascending order: 1 1 3 0, 1 6, 3, 2, 5, 1 The median is the mean of the two middle terms, since there is an even number of terms in the set: 1 5 +1 5 6 median = 3 2 2 = 2 = 12 Next, nd the mean: 1 1 3 30 5 78 + 10 + 15 + 18 0+ 1 78 6 + 3 + 2 + 5 +1 30 + 30 mean = = 30 30 30 = 30 6 6 6 = 180 For ease of comparison, nd a common denominator: 5 15 75 12 15 = 180 78 75 3 1 The dierence is 180 180 = 180 = 60 , choice (D). 8. B Explanation: There are three possible solutions to the equation; each one satises one of the following: y+4=0 y 7=0 4y + 1 = 0 1 That means the possible values of y are -4, 7, and 4 . The latter two are in the set, but the rst is not. Therefore, of the 12 numbers in the set, two desired of them would be a solution to the equation. Since probability is possible , the 2 1 answer is 12 = 6 , choice (B). 9. D Explanation: If the standard deviation is 2.5, 2.5 standard deviations is the 2:5(2:5) = 6:25. So, we want to know which of the numbers is NOT more than 6.25 away from the mean of 30.0. That would be any number not less than 30 6:25 = 23:75 or any number greater than 30 + 6:25 = 36:25. The only number that is within that range is (D). 10. D Explanation: To nd one of the endpoints, set up the overlapping sets formula as if 40 of the students (the minimum number) were neither born nor attended high school in the state: 30 Copyright 2008 Je Sackmann jesackmann@gmail.com - www.gmathacks.com

6. EXPLANATIONS

Je Sackmann GMAT Tutor jesackmann@gmail.com www.gmathacks.com

T = G1 + G2 B + N 300 = 210 + 250 x + 40 300 = 500 x x = 200 That limits our choices to (D) and (E). The number that t into both categories is constrained by the size of each category: since there are only 210 students who were born in the state, only 210 students can both be born in the state and have attended high school in the state. Thus, (D) is correct. 11. D Explanation: If the average price of three frames is $18, the total price for the three frames is 3(18) = 54. If the average price of 4 frames is $20, the total cost of the four frames is 4(20) = 80. The fourth frame must cost the dierence: 80 54 = 26, choice (D). 12. B Explanation: The numbers are already arranged in ascending order, so to nd the median, nd whether the number of terms is even or odd (it s even) and then nd the mean of the two middle terms: 25+30 = 55 2 2 = 27:5 The mean requires more work. Add the terms and divide by 10: 15+18+20+24+25+30+50+50+64+80 = 376 10 10 = 37:6 Only one term, $30, is less than 37.6 and more than 27.5. The correct choice is (B). 13. A Explanation: When only the most and least expensive amounts are changed, the middle term is unaected, so the median stays the same. That leaves us (A) and (C). To determine whether the mean changes, see if the total price of the homes changes. Since the most expensive house increases by the same amount that the least expensive house decreases, the total price stays the same. Thus the mean does, as well. (A) is the correct answer. 14. E Explanation: The mean of 40, 60, and 80 is: 40+60+80 = 60 3 60 is 5 more than 55, so 55 is the average of 30, 50, and the answer, which we can call x: 30+50+x = 55 3 30 + 50 + x = 165 x = 85, choice (E). 15. E

31 Copyright 2008 Je Sackmann jesackmann@gmail.com - www.gmathacks.com

6. EXPLANATIONS

Je Sackmann GMAT Tutor jesackmann@gmail.com www.gmathacks.com

Explanation: 8 standard deviations is $0:04(8) = $0:32, while 9 standard deviations is $0:04(9) = $0:36. 8 and 9 standard deviations above the mean, then is: $0:16 + $0:32 = $0:48 $0:16 + $0:36 = $0:52 The only choice between those two values is (E), $0.49. 16. B Explanation: The 100-page document would cost $50 for the rst 20 pages, leaving 80 more pages. That s 8 10-page segments, each of which cost $15, for a total of 8(15) = 120. The total, then, is 50 + 120 = 170. The average charge per page is the total charge divided by the number of pages: 170 = 1:70, choice (B). average = 100 17. D Explanation: Since the median is 50, an equal number of terms must be greater than 50 and less than 50. That means one of the terms is 50, and 12 are less than 50. Since none of the terms are the same and we want the largest possible integer in the set, the numbers less than 50 should be consecutive, so that the smallest number in the set is as large as possible. If all the numbers are consecutive from 50 down to the least integer, the bottom 13 numbers are as follows: 38, 39, 40, 41, 42, 43, 44, 45, 46, 47, 48, 49, 50 If the lowest number is 38 and the range is 50, the greatest number is 38 + 50 = 88, choice (D). There is no way for the set to have a larger smallest number, so there s no way for the greatest number to be any larger, either. 18. B Explanation: To nd the median, arrange the terms in order: 72, 72, 72, 76, 76, 76, 78, 89, 89, 93, 93 Since there is an odd number of terms, the middle number is the median. There are ve terms below or equal to 76, and ve terms greater than 76, so that s our median, and choice (B) is correct. 19. B Explanation: If every term is increased or decreased by the same amount, the dispersal of the set is unchanged. The standard deviation depends on how far each term is from the mean, and subtracting 5 from every term doesn t change that. Thus, m is unchanged, and (B) is the correct answer. 20. B Explanation: If 4 is the greatest integer in a series of 15 consecutive integers, the smallest number is 4 14 = 10. (Subtract 14 because the dierence between the least and greatest of x consecutive integers is x 1.) The negative integers in the set, then, run from 10 up to 1. The range of those is given by: 32 Copyright 2008 Je Sackmann jesackmann@gmail.com - www.gmathacks.com

6. EXPLANATIONS

Je Sackmann GMAT Tutor jesackmann@gmail.com www.gmathacks.com

( 10) = 9, choice (B).

21. B Explanation: like this:

Technically, a weighted average problem should be set up

However, it s easier to subtract 15,000 from each. (Actually, you can do even better, and forget about the thousands.) The result looks like this: 0(3)+2(1)+3(2) 1 =8 6 6 = 13 Add 15 back to the result: 1 11 3 + 15 = 16 3 Since we re measuring in thousands, that s about the same asa $16,300, choice (B). 22. D Explanation: Since the list is in increasing order, we can nd the median. There is an even number of terms, so the median is the mean of the middle two: 5+7 12 2 = 2 =6 The median, 6, is 2 3 of the mean: 2 6= 3 (mean) 3 mean = 6( 2 )=9 Now that we know the mean, we can solve for x: 2+4+5+7+15+x =9 6 33 + x = 54 x = 21, choice (D). 23. D Explanation: If a GPA of 2.8 is 1.5 standard deviations below the mean, we can devise an equation, in which m is the mean and d is standard deviation: 2:8 = m 1:5d 3.6 is 2.5 standard deviations above the mean: 3:6 = m + 2:5d The easiest way to solve for one of the variables is to subtract the rst equation from the second: 3:6 2:8 = (m m) + (2:5d ( 1:5d)) 0:8 = 0 + 4d d = 0:2 We can use the value of d to nd the mean in the rst equation: 2:8 = m 1:5(0:2) 2:8 = m 0:3 m = 3:1, choice (D). 24. C Explanation: equal to 17:

15;000(3)+17;000(1)+18;000(2) 6

Add the terms together, divide by 4, and set the result = 17

k+(2k 5)+(3k+2)+(6k 1) 4

33 Copyright 2008 Je Sackmann jesackmann@gmail.com - www.gmathacks.com

6. EXPLANATIONS

Je Sackmann GMAT Tutor jesackmann@gmail.com www.gmathacks.com

= 17 12k 4 = 68 12k = 72 k = 6, choice (C). 25. D Explanation: Since we don t know the standard deviation, the only way to add terms and guarantee that the standard deviation will decrease is to add terms that are equal to the mean. The standard deviation is approximately the average dierence of terms from the mean. If a term is equal to the mean, its variance from the mean is 0. So if we add 85 and 85, choice (D), the standard deviation must get smaller, since we know that the deviation is currently positive. 26. D Explanation: Since the question suggests that the answer is always the same, it is probably most e cient to choose a value for z and evaluate each of the expressions. 1 s say 2 is usually a good bet for values between 0 and 1, so let z=1 : 2 z=1 2 z 1= 1 q 2 p 1 1 1 5 p z= 1 2 = 1 :4 = 7 = 7 2
1 2 z2 = ( 2 ) =1 4 1 3 3 z = (2) = 1 8 In order, we have to z 2 , choice (D).
5

12k 4 4

1 1 1 1 2, 8, 4, 2,

1 and 5 7 . The middle term is 4 , which is equal

27. D Explanation: In order to maximize the smallest weight, we need to minimize the other two weights. Since the median is 180, one of the weights must be 180. The weight that is to the right of 180 in an ascending list can be equal to 180, so to minimize it, let s say it is 180 as well. Since the mean is 160, the total weight is 3(160) = 480. If both of the heavier weights are 180, the lightest of the 3 must be: 480 180 180 = 120, choice (D). 28. C Explanation:
26:5(30)+25:5(20) 50

Technically, a weighted average should be set up like this:

However, the math is much simpler if we subtract 25.5 from each of the terms, remembering to add it back to the result later on: 1(30)+0(20) = 30 50 50 = 0:6 Our answer, then, is 0:6 + 25:5 = 26:10, choice (C). 29. A 34 Copyright 2008 Je Sackmann jesackmann@gmail.com - www.gmathacks.com

6. EXPLANATIONS

Je Sackmann GMAT Tutor jesackmann@gmail.com www.gmathacks.com

Explanation: A value two standard deviations from the mean is 2(1:75) = 3:5 away from the mean. We don t know whether our answer is 3.5 less or 3.5 greater, so let s nd both: 9:5 + 3:5 = 13 (not a choice) 9:5 3:5 = 6, choice (A). 30. D Explanation: First, multiply out each of the expressions, since we ll have to add them to nd the average: (x + 2)2 = x2 + 4x + 4 (x + 2)(x 2) = x2 4 Now nd the average: 2 (x2 +4x+4)+(x2 4) +4x = 2x 2 = x2 + 2x, choice (D). 2 31. B Explanation: average is:
W p ounds s

Average is given by sum over number, so in pounds, the

However, we re looking for the average in ounces, so we must convert given the ratio in the question: W p ounds 16 ounces 16W ounces , choice (B). s 1 p ound = s 32. B Explanation: If 40% of the employees have passed the exam, 60% have not. Those 60% comprise a total of 32+16 = 48 employees, so the total number of employees at the rm is given as follows: 0:6e = 48 6 10 e = 48 e = 48( 10 6 ) = 8(10) = 80, choice (B). 33. A Explanation: There s only one variable, so we can set up the average formula and solve for y: 32+(y +2)+(y +5)+30+y = 24 5 62 + 3y + 7 = 24(5) 69 + 3y = 120 3y = 51 y = 17, choice (A). 34. B Explanation: To nd the least possible number for one of the students, we must maximize the other two numbers of hours. If the upper limit on the number of hours is 10 hours, let s say each of the other two students spent the full 10 hours. Now, we can use the average formula to nd the number of hours that the third student must have spent in order for the three to average 7.5 hours: 35 Copyright 2008 Je Sackmann jesackmann@gmail.com - www.gmathacks.com

6. EXPLANATIONS

Je Sackmann GMAT Tutor jesackmann@gmail.com www.gmathacks.com

x 7:5 = 10+10+ 3 22:5 = 20 + x x = 2:5, choice (B).

35. C Explanation: To nd the median, start by arranging the terms in ascending order: 0:25, 0:10, 0:05, 0:05, 0:25, 0:60 Since there is an even number of terms, the median is the mean of the middle two terms: :05 median = 0:05+0 =0 2 2 = 0, choice (C). 36. D Explanation: First, nd the total number of pieces of luggage: 50(2) + 40(1) + 10(3) + 10(4) = 100 + 40 + 30 + 40 = 210 The number of pieces of luggage per passenger is that total number of pieces divided by the number of passengers: 21 7 210 120 = 12 = 4 = 1:75, choice (D). 37. D Explanation: Since the mean is 80, numbers within 1 standard deviation (6, in this case) of the mean must be between the following two numbers: 80 + 6 = 86 80 6 = 74 6 of the 8 numbers are between 74 and 86: 6 3 8 = 4 = 75%, choice (D). 38. B Explanation: The total number of pilots is the sum of the number who y the individual routes, minus the number of overlaps, as those indicate that those pilots were counted twice in the table: 35 + 25 + 20 (7 + 4 + 3) = 80 14 = 66, choice (B). 39. E Explanation: In April, if the range is $22,000 and the lowest price is $184,000, the highest price is $184; 000 + $22; 000 = $206; 000. In April, the range is $25,000 and the lowest is $192,000, so the highest price is $192; 000 + $25; 000 = $217; 000. Overall, the lowest price is $184,000 and the highest is $217,000, for a range of $217; 000 $184; 000 = $33; 000, choice (E). 40. A Explanation: The greatest possible average price relies on the greatest possible prices for the nal two machines, which are each $9,000. This average is, then: 36 Copyright 2008 Je Sackmann jesackmann@gmail.com - www.gmathacks.com

6. EXPLANATIONS

Je Sackmann GMAT Tutor jesackmann@gmail.com www.gmathacks.com

000 = 32;4 = 8; 000 The least possible price relies on the least possible prices for the nal two machines, or $5,000 each: 5;000+9;000+5;000+5;000 000 = 24;4 = 6; 000 4 The dierence between the least and greatest average prices is 8; 000 6; 000 = 2; 000, choice (A). 5;000+9;000+9;000+9;000 4

41. B Explanation: If each of the terms is multiplied by the same number, the dierences in the terms is multiplied by the same amount. For instance, if the terms were initially 1, 2, and 3, the new terms are 3, 6, and 9 the numbers tripled, and the dierences between them tripled as well. While this is only an approximation, it s enough to know that the standard deviation changes by the same factor, so the new standard deviation is 3z , choice (B). 42. D Explanation: In order to meet the goal, the retailer must sell 250; 000 170; 000 = 80; 000 in the nal 40 days. As a per day average, that is 80;000 = 8;000 = 2; 000, choice (D). 40 4 43. C Explanation: Before we can determine what is 1 standard deviation away from the mean, we must calculate the mean: 35+37:5+40+42:5+45+52:5+52:5+55+60+60 = 480 10 10 = 48 One standard deviation is plus or minus 9.2: 48 + 9:2 = 57:2 48 9:2 = 38:8 The terms outside of that range are 35, 37.5, 60, and 60, a total of four, choice (C). 44. A 1 Explanation: 3 of those who own their own home (153 people) have children, so the overlap between the two sets is 1 Since 80 have 3 (153) = 51. children and 51 of those own their own home, 29 do not. If one person is randomly selected from the group of 232, there are 232 possible outcomes, and 29 desired outcomes: 29 probability = 232 =1 8 , choice (A). 45. C Explanation: The range is the dierence between the lowest and highest terms. In this case, those terms are 6 and 16, so the range is 16 6 = 10, choice (C). 46. B Explanation: A decrease is a negative increase, so the three increases are 15,000, 25,000, and -17,500. The average, then, in thousands, is 37 Copyright 2008 Je Sackmann jesackmann@gmail.com - www.gmathacks.com

6. EXPLANATIONS

Je Sackmann GMAT Tutor jesackmann@gmail.com www.gmathacks.com

:5 = 22 3 = 7:5 Converted back to thousands of dollars, the average is $7,500, choice (B). 15+25 17:5 3

47. D Explanation: To maximize the number of classes attended by a single member, minimize the number by everyone else that you can. 16 attended 0, 1, or 2 classes, meaning that 4 attended 3 or more classes. Since one of those is the member we re trying to maximize, say that three of those remaining members attended 3 classes each. That gives us a total number of classes of 3(0) + 8(1) + 5(2) + 3(3) + 1(x) = 8 + 10 + 9 + x = 27 + x If the average number per member is 1.75 and there are 20 members, the total number of classes attended by this sample is 20(1:75) = 35. We can solve for x: 35 = 27 + x x = 8, choice (D). 48. B Explanation: First, nd the median by arranging the terms in ascending order: 28; 39; 40; 51; 52 The middle term is 40, and since the number of terms is odd, that s the median. Next, nd the average: 28+39+40+51+52 = 210 5 5 = 42 The dierence is 42 40 = 2, choice (B). 49. B Explanation: If the three people called an average of three times, the total number of calls was 3(3) = 9. If Hank called 5 times and Jelena called 2 times, that leaves 2 times unaccounted for, so that must be the number of times that Kristof called. The correct answer is (B). 50. E Explanation: The weighted average formula allows us to set up this problem like this (ignoring the thousands for simplicity): 6(14)+5(16)+4(x) = 16 15 To make the math easier, subtract 14 from each selling price, and add it back later: 6(0)+5(2)+4(x) =2 15 10 + 4x = 30 4x = 20 x=5 Add 14 back to the total: 5 + 14 = 19 Convert to thousands of dollars, and you have your answer, $19,000, choice (E).

38 Copyright 2008 Je Sackmann jesackmann@gmail.com - www.gmathacks.com

6. EXPLANATIONS

Je Sackmann GMAT Tutor jesackmann@gmail.com www.gmathacks.com

51. C Explanation: The median of list I is the mean of the middle two numbers: 8+10 18 = = 9 2 2 Since list II has an odd number of terms, the median must be one of the terms. Since the median must be 9, and 9 isn t one of the integers listed in II, k must be 9, choice (C). 52. D Explanation: The average is the sum of terms (conference calls, in this case) divided by the number: 5+7+8+5+12 = 37 5 5 = 7:4, choice (D). Set up the weighted average equation as follows: = 4; 000 Make the math simpler by subtracting 3,000 from each dollar amount and add it back to the value of x later: 16(0)+4x = 1; 000 20 4x = 20; 000 x = 5; 000 Add 3,000 back: 5; 000 + 3; 000 = 8; 000, choice (E).
16(3;000)+4(x) 20

53. E Explanation:

54. D Explanation: The average of 10, 15, and 20 is 45 10+15+20 = = 15 3 3 If 15 is the average of 12, 18, and the answer, we can solve for the variable: x 15 = 12+18+ 3 45 = 30 + x x = 15, choice (D). 55. B Explanation: This is a perfect example for the overlapping sets equation: T = G1 + G2 B + N 18 = 10 + 12 B + 3 18 = 25 B B = 7, choice (B). 56. B Explanation: If the average of a and b is 60, the sum of the terms is 120. Similarly, the sum of b and c is 70(2) = 140. That gives us two equations: b + c = 140 a + b = 120 To nd c a, subtract the second equation from the rst: c + b (a + b) = 140 120 c a = 20, choice (B).

39 Copyright 2008 Je Sackmann jesackmann@gmail.com - www.gmathacks.com

6. EXPLANATIONS

Je Sackmann GMAT Tutor jesackmann@gmail.com www.gmathacks.com

57. D Explanation: This question is tailor-made for the overlapping sets formula. Remember, when dealing with percents, that the total is always 100. T = G1 + G2 B + N 100 = 65 + 65 B + 25 100 = 155 B B = 55, choice (D). 58. C Explanation: The total number of sales Joan made over the past x days is 15x, so after making 25 sales today, her total is 15x + 25. If her average is 16 sales over thex + 1 days, her total can also be expressed as 16(x + 1). To solve for x, set the two expressions for the total equal to each other: 15x + 25 = 16(x + 1) 15x + 25 = 16x + 16 x = 9, choice (C). 59. C Explanation: Another way of phrasing this question is like so: "How many positive even integers less than 75 have square roots that are also positive even integers less than 75?" Or, simpler still: "How many positive even integers less than 75 are perfect squares?" There are 4 of those: 4, 16, 36, and 64, which means that choice (C) is correct. 60. D Explanation: The remaining pieces, after removing a 16-foot piece, have a total length of 40 16 = 24 feet. There are 4 pieces left, so the average is 24 4 = 6, choice (D). 61. D Explanation: Statement (1) is su cient: set up the equation and simplify: p+q +8 =p 3 p + q + 8 = 3p q = 2p 8 If p is an integer, 2p is also an integer, and 2p 8 is an integer as well. Therefore, q must be an integer. Statement (2) is su cient: again, set up the equation and simplify: p+q +3:5 = 3:5 3 p + q + 3:5 = 10:5 q=7 p If p is an integer, then q is equal to an integer (7) minus an integer (p), so it must be an integer as well. Choice (D) is correct.

40 Copyright 2008 Je Sackmann jesackmann@gmail.com - www.gmathacks.com

6. EXPLANATIONS

Je Sackmann GMAT Tutor jesackmann@gmail.com www.gmathacks.com

62. A Explanation: In a set of numbers, the mean is equal to the median if the numbers are equally spaced, as in consecutive integers, consecutive evens, or consecutive multiples of 5. Statement (1) is su cient: it establishes that the numbers in the set are consecutive, so they are equally spaced, so the mean and the median are equal. Statement (2) is insu cient: it doesn t matter what the rst number is; if the rst number is odd, the numbers could be equally spaced, or they could not be. The correct choice is (A). 63. C Explanation: Statement (1) is insu cient: while they are consecutive, we don t know what order they re in. Since the median is the middle number, that stands in the way of determining the median. Statement (2) is also insu cient. In a set of 5 numbers, the mean is not necessarily the median: a set like {-1, 5, 6, 7, 8} has a mean of 5, which is part of the set, but not the median of the set. Taken together, the statements are su cient. In a set of consecutive integers, the mean and the median are equal. Thus, if d is the mean, d is also the median. (C) is the correct choice. 64. E Explanation: Statement (1) is not su cient: in order to answer the question, we d also need to know how many bought their ticket at least two weeks in advance and paid full price. Statement (2) is also insu cient. To answer the question, we d need to know how many did not buy their ticket two weeks in advance and did not pay full price that would give us the total number who did not buy their ticket two weeks in advance, from which we d know how many did. Taken together, the statements are still insu cient. For each choice, I ve explained what information is lacking neither statement provides the information that the other one requires to solve for the answer. Choice (E) is correct. 65. D Explanation: Since all 120 clients have health insurance or drug coverage, there are only three subgroups: those who have insurance only, those who have drug coverage only, and those who have both. The 40 who do not have drug coverage must then have insurance only. Statement (1) is su cient: if 92 have insurance and 40 have insurance only, that leaves 52 who have both. Statement (2) is also su cient. The 28 who don t have insurance must have drug coverage only. Thus, of the 120 total, 28 have drug coverage only, 40 have insurance only, which leaves 120 48 20 = 52 who have both. Choice (D) is correct. 66. E

41 Copyright 2008 Je Sackmann jesackmann@gmail.com - www.gmathacks.com

6. EXPLANATIONS

Je Sackmann GMAT Tutor jesackmann@gmail.com www.gmathacks.com

Explanation: Statement (1) is insu cient. If two of the numbers are greater than 60, at least one of the numbers must be less than 60, but it s possible that none, one, or two of the remaining numbers are equal to 60. Statement (2) is also insu cient. If two of the numbers are less than 60, at least one of the numbers must be greater than 60, but it s possible that none, one, or two of the remaining numbers are equal to 60. Taken together, the statements are still insu cient. A maximum of one number could be equal to 60, but just because the mean of a set is 60 doesn t mean that any of the terms are equal to 60. The set could be {50, 50, 60, 70, 70}, but it could also be {50, 50, 58, 70, 72}, in which case the answer is dierent. Choice (E) is correct. 67. E Explanation: First, deal with the information you re given. If the largest number of set S is v and the range is p, the smallest number in set S is v p. Similarly, the smallest number is set T is w-q. The question is asking: "Is v p > w q ?" Statement (1) is insu cient: that relates to the question, but it tells us nothing about v and w, which are equally important. Statement (2) is also insu cient. The mean and median are all but useless when it comes to nding the extremes in a set. They tell us nothing about the values of the ranges or largest or smallest numbers. Taken together, the statements are insu cient. (2) doesn t tell us anything of substance, and (1) doesn t address the size of the largest terms in each set. Without that piece of information, we can t solve. Choice (E) is correct. 68. D y +z , so in Explanation: The average of the three variables is equal to x+3 order to nd the average, we only need the sum of the three terms. Statement (1) is su cient: multiply both sides by 2 and the sum of the three terms is 18. Statement (2) is also su cient: it gives us the average formula directly, so the sum of the terms is 18 and the average, which is given, is 6. Choice (D) is correct. 69. A Explanation: Statement (1) is su cient. The rst ten positive even integers extend from 2 to 20, inclusive. In a set of consecutive terms, the mean and median are equal, and are equal to the average of the least and greatest terms. So, the mean of the set is 2+20 = 11. If z < 11, z is less than 16. 2 Statement (2) is not su cient. There are an innite number of squares of integers, some of which are less than 16 (1, 4, and 9) and many more than are not less than 16 (25, 36, 49, etc.). Choice (A) is correct. 70. C

42 Copyright 2008 Je Sackmann jesackmann@gmail.com - www.gmathacks.com

6. EXPLANATIONS

Je Sackmann GMAT Tutor jesackmann@gmail.com www.gmathacks.com

Explanation: We know how many drive to work and are more than 30 years old; to nd how many drive to work and are under 30, we need to know the total number who drive to work. Statement (1) is insu cient: we could use to nd the number who are 30 years old or more and do not drive to work, but that s not what we re looking for. Statement (2) is also insu cient: we can t combine that information with what s given to nd out anything specic. Taken together, the statements are su cient. (1) tells us how many are 30 years old or more and do not drive to work; (2) tells us how many are under 30 and do not drive to work, so we can nd the total number who do not drive to work. Since we know the total, we can nd the total number who DO drive to work which, as we established earlier, is enough to nd the subgroup we re looking for. Choice (C) is correct. 71. E Explanation: Statements (1) and (2) both tell you similar things: e is greater than the sum of two of the terms. Since all the numbers are positive, e must be greater than all of those terms that are added together. Thus, e is the largest of the ve numbers. However, even taken together, that tells us nothing about how the other four numbers are arranged: the set could consist of {1, 2, 3, 4, 10}, but we don t know which of {1, 2, 3, 4} is which variable. Choice (E) is correct. 72. A Explanation: Statement (1) is su cient. If p is four times greater than n and three times greater than q , the numbers line up like this: p>q>n So, q is the median. Statement (2) is insu cient: we need to know something about the other two variables, as well. 73. D Explanation: Since the integers are consecutive evens, if we can nd out one of the numbers the least or the greatest, or any term that gives us those we ll know all we need to know about the entire set. Statement (1) is su cient. If the average of the rst eight integers is 13, we can nd what those rst 8 integers are. As it turns out, they are the evens from 6 to 20, so the twelve integers are the evens from 6 to 28. Statement (2) is also su cient. If the average of the last eight is 21, we can gure out that those eight are the evens from 14 to 28. That means that entire set of 12 evens stretches from 6 to 28. Choice (D) is correct. 74. E Explanation:

To nd the average, we need the sum of the three terms.

43 Copyright 2008 Je Sackmann jesackmann@gmail.com - www.gmathacks.com

6. EXPLANATIONS

Je Sackmann GMAT Tutor jesackmann@gmail.com www.gmathacks.com

Statement (1) is insu cient: there s nothing about c. (2) is also insu cient, since we don t know anything about a. Taken together, the statements are still insu cient. We have two linear equations and three variables, which isn t enough to nd the values of each of the variables. There s no other way to nd a + b + c, so choice (E) is correct. 75. E Explanation: Statement (1) is insu cient. We know that 72 represents 15% of the total, so we can nd the total, but there s no way to compare the number with sales experience to the number with management experience, since all we know is the number that have both. Statement (2) is also insu cient. The number who have neither is irrelevant to a comparison of how many have each type of experience. Taken together, the statements are still insu cient. If 72 is 15% of the total, there are 480 employees in the company, which means that 480 252 = 228 have either management or sales experience, or both. Since 72 have both, that means 156 have one or the other, but not both. However, we have no way of determining how many have each kind, so (E) is the correct answer. 76. B Explanation: Statement (1) is insu cient. Standard deviation is based on the exact measurements. So knowing the total increase is not enough: the standard deviation would be dierent if that 17 inches was entirely due to the growth of one child than if the 17 inches was evenly distributed among the six children. Statement (2) is su cient. If each term in a set increases by the same percent, the standard deviation increases by that percent as well. We know the standard deviation two years ago, and we know how many each individual term changes (in percent terms), so we can calculate the new standard deviation. Choice (B) is correct. 77. E Explanation: Statement (1) is insu cient. We know in how many ponds the population of sh decreased, but not in how many of those the population of bacteria did not decrease. Statement (2) is also insu cient: If both increased or stayed constant in 18, that leaves 12 ponds in which either the population of sh, of bacteria, or both decreased. However, that s not enough to answer the question in how many ponds the sh population decreased and bacteria didn t. Taken together, the statements are insu cient. 7 is the total of the subgroups "sh decrease" and "both decrease", while 12 is the total of those two, plus "bacteria decrease." Thus, 5 ponds have "bacteria decrease," but again, that s not what we re looking for. We don t have the information to separate the two subgroups of the total of 7, so the correct choice is (E). 78. C 44 Copyright 2008 Je Sackmann jesackmann@gmail.com - www.gmathacks.com

6. EXPLANATIONS

Je Sackmann GMAT Tutor jesackmann@gmail.com www.gmathacks.com

Explanation: For each of the two teams, the height of the tallest member is the height of the least member plus the range. Team A: j + a Team B: k + b So, the question is asking: "Is j + a < k + b ?" Statements (1) and (2) both fail to provide enough information: to answer the question, we need something about all four variables, and each statement only oers something about two each. Taken together, the statements are su cient. If j < k and a < b, then the sum j + a must be less than k + b. That s what the question wants to know, so (C) is correct. 79. A Explanation: Statement (1) is su cient. If the average of six of the numbers is 72, and the average of the seven numbers is 72, the seventh number must be 72. To see why, consider that if the average of six of the numbers is 72, the total of those numbers is 6(72). So, to solve for the nal number, use the average formula: 6(72)+x = 72 7 6(72) + x = 7(72) x = 7(72) 6(72) = 72 Since the other six are odd integers, none of those can be equal to 72. One number is equal to 72. Statement (2) is insu cient. We re only given information about one of the numbers, and the possibility is left over that many more are also equal to 72. (A) is the correct choice. 80. E Explanation: The overlapping sets formula is a handy way to work with this question. If Group 1 is the number of blondes, and Group 2 is the number of males: T = G1 + G2 Both + N either Each of statement (1) and (2) only give us one of those variables each, and the question only gives us one as well. In a ve-variable equation, two variables isn t going to give us enough information. Taken together, the statements are still insu cient. Plug in the information given: 56 = 36 + 32 Both + N either Both = 12 + N either Unless we know how many people in the group are neither blonde nor male, we won t know how many are blonde and male. Choice (E) is correct. 81. E Explanation: Statement (1) is insu cient. It helps a little bit: it reduces the four subgroups (willing to travel only, specialize in marketing only, both,

45 Copyright 2008 Je Sackmann jesackmann@gmail.com - www.gmathacks.com

6. EXPLANATIONS

Je Sackmann GMAT Tutor jesackmann@gmail.com www.gmathacks.com

and neither) to three, since none are only willing to travel to City X. Still, there are three subgroups and we know nothing but the total of the three. Statement (2) is also insu cient. In terms of the subgroups listed in (1), it tells us that 5 are "neither." Of the four subgroups, that leaves us with three unknowns. Taken together, the statements are still insu cient. 32 is the total of three subgroups: those who specialize in marketing but are not willing to travel to City X (which is what we re looking for), those who do both, and those who do neither (5). We re left with two variables, so we can t solve for our target. Choice (E) is correct. 82. E Explanation: Statement (1) tells you something you already know. If the average is 16, the sum of the two terms is 32. That s redundant, so not only is (1) insu cient, but the statements combined won t be su cient (if we get that far), because (1) doesn t contribute any information at all. Statement (2) is also insu cient. The positive dierence means that 16 could be equal to v z or z v . A dierence of 16 and a sum of 32 means that the numbers are 24 and 8, but the answer to the question is dierent depending on which variable is which of the two numbers. As we saw with (1), there s no purpose in combining the statements in this example, so choice (E) is correct. 83. C Explanation: Each of the statements is insu cient on its own. Each tells you that the gap between two of the numbers is 4. If the other two terms are between those endpoints, the range of the set could be as little as 4. But since we don t know, in either case, about the other two terms, they could be much more dispersed, resulting in a range larger than 6. Combined, the statements are su cient. Add the equations together: w x=4 x z=4 w z=8 Since the dierence between w and z is 8, the range of the set must be at least 8. It could be larger, if y is not between the 2, but regardless of whether that s the case, the range is at least 8. Choice (C) is correct. 84. B Explanation: Converting the question to algebra: 0 < x < 10 ? z < x+10 2 Statement (1) is insu cient. Combined with the equation in the question, this means that z is between 0 and 60. Sometimes that s less than the average 1 of z and 10 (if z = 1, x = 6 , and the average of x and 10 is about 5), but most of the time it s not (if z = 24, x = 4, average of x and 10 is 7). Statement (2) is su cient. This could mean one of two things. If z is greater than 10, this is always true, since a number greater than 10 is always 46 Copyright 2008 Je Sackmann jesackmann@gmail.com - www.gmathacks.com

6. EXPLANATIONS

Je Sackmann GMAT Tutor jesackmann@gmail.com www.gmathacks.com

closer to 10 than it is to a number that is less than 10, such as x. It could also mean, if z is less than 10, that 10 z < z x. In the rst case, z is always greater than the average of x and 10. In the second case, it is also greater. The average of x and 10 is the halfway point between those two numbers. If z is closer to 10 than to x, it is greater than the halfway point, so it is greater than the average. Choice (B) is correct. 85. D Explanation: As always with average, we can nd the answer if we can nd the sum of the two terms. (To nd the average, we d divide that sum by 2.) Statement (1) is su cient. Set up the equation: (j +3)+(k+3) = 12 2 j + k + 6 = 24 j + k = 18 Statement (2) is also su cient. Again, set up the equation and try to isolate j + k: j +k+6 =8 3 j + k + 6 = 24 j + k = 18 Choice (D) is correct. 86. E Explanation: Statement (1) is insu cient. If 115 are made of leather, 85 are not made of leather. If 65 of those are not designed for men, 20 of them are designed for men. That s doesn t answer the question, though. Statement (2) is also insu cient. In fact, it repeats what the other statement tells us, since we solved for that data point in (1). If 20 of those designed for men are not made of leather and 85 are not made of leather, that means 65 are not made of leather and not designed for men. Taken together, we don t gain anything. The statements tell us the same thing, and since each is insu cient, combined they are also insu cient. (E) is correct. 87. E Explanation: Aside from the two unknowns, the endpoints of the set are 4 and 7. The range will be greater than 9 only if one of three scenarios is true: - The dierence between x and y is greater than 9. - x or y is greater than 13. - x or y is less than -6. Statement (1) is insu cient. If x is 0, all it means is that y must be positive. The numbers could be very close together, and between -6 and 13. However, it s a weak limitation: even if x = 0, y could be anything, including numbers much greater than 13. Statement (2) is also insu cient. Again, the variables could be very large numbers, but they could also be 1 and 2, in which case the range is less than 9. 47 Copyright 2008 Je Sackmann jesackmann@gmail.com - www.gmathacks.com

6. EXPLANATIONS

Je Sackmann GMAT Tutor jesackmann@gmail.com www.gmathacks.com

Taken together, the statements are still insu cient. In both statements, one or both of the variables could be very large, in which case the range is greater than 9. However, if x = 1, y could be as small as 5. If those are the values of x and y , then the range is 6, which is less than 9. Choice (E) is correct. 88. E Explanation: Statement (1) is insu cient. Since all the integers are positive, we do know that n is greater than either q or r. However, we need to know about the other two terms, as well. Statement (2) is also insu cient. It doesn t give us any information about the other three terms. Taken together, the statements are still insu cient. Any of the ve terms could be the median: for instance, if m and p are both greater than n, n is the median. However, if m and p are both less than q and r, either q or r coudl be the median. (E) is the correct choice. 89. E Explanation: Statement (1) is insu cient: all or none (or one or two) of the other numbers could also be equal to 100 and still allow the average to be 100. Statement (2) is also insu cient. If one is 50, at least one must be greater than 100, but that leaves the options of zero, one, or two of the numbers to be equal to 100. Taken together, the statements are insu cient. If the average is 100, the sum of the 4 terms is 400. That means the sum of the remaining two terms is 250. One of those terms could be 100, in which case the other is 150; also, it s possible that both could be 125, among many other possibilities. There could be one or two total terms that are equal to 100. Choice (E) is correct. 90. A Explanation: Statement (1) is su cient. If John s average score was 112, his total scores were 112 times 3. If the average of his highest and lowest were 118, the sum of those two is 2 times 118. We don t need to do the math, but with those numbers, we could nd his middle score it s the dierence between the total of all 3 and the total of the highest and lowest. Since the median is, by denition, the middle score, the third game is the median. Statement (2) is insu cient. Between the average score and his highest score, we can nd the sum of his lowest and middle scores, but in order to nd the median, we need the exact amount of his middle score. Choice (A) is correct. 91. E Explanation: Statement (1) is insu cient. In fact, it s irrelevant. We don t need to know anything about the total population or how the number 25 years olds relates to it.

48 Copyright 2008 Je Sackmann jesackmann@gmail.com - www.gmathacks.com

6. EXPLANATIONS

Je Sackmann GMAT Tutor jesackmann@gmail.com www.gmathacks.com

Statement (2) is also insu cient. This is more helpful, though. There are two subgroups of 25 year olds (men and women), and the subgroups are 90% and 75% full-time students, respectively. If there is an equal number of men and women, that means 82.5% of those 25 years old and younger are full-time students. However, we don t know whether there are equal numbers of men and women. If there are enough men, that drags the average down below 80% and gives us a dierent answer to the question. Taken together, the statements are still insu cient. (1) doesn t help us gure out what fraction of those 25 and younger are men or women, which is what we need to add to (2) to answer the question. (E) is correct. 92. E Explanation: Statement (1) is insu cient: we don t know anything about the populations of the AA and A rated countries. Statement (2) is also insu cient, as it doesn t tell us anything about the populations of the AAA rated countries. Taken together, the statements are still insu cient. 25 percent of the AAA countries have populations greater than 20 million, and 30 percent of the AA and A countries have populations greater than 20 million. For an overall percentage, though, we d need to know the numbers (or at least the ratio) of AAA and AA/A countries. While we know that the percent is between 25 and 30 percent, we can t nd the exact percent without that ratio. (E) is the correct choice. 93. C Explanation: Statement (1) is insu cient. Between the question and this statement, we know how much the sets overlap, but we don t know anything about the actual size of the sets, which is very important when determining how many unique integers are in the three sets. Statement (2) is also insu cient. We know the total number in each set, and the number of overlaps in two sets, but we don t know how many terms are in all three sets. Taken together, we have enough information. Given the size of the sets, the overlaps between each pair of sets, and the overlap of all three sets, we can nd the total number of integers. Choice (C) is correct. 94. A Explanation: Statement (1) is su cient. 60% of biotech employees are biotech executives, and 75% of the total are biotech employees. So, the percent of the total that is biotech executives is 60% of 75% of the total, or (0:6)(0:75) = 0:45. Statement (2) is insu cient. We re looking for a percent. We don t need a number, and furthermore, this number isn t relevant to the specic subgroup we re looking for. Choice (A) is correct. 95. C 49 Copyright 2008 Je Sackmann jesackmann@gmail.com - www.gmathacks.com

6. EXPLANATIONS

Je Sackmann GMAT Tutor jesackmann@gmail.com www.gmathacks.com

Explanation: Statement (1) is insu cient. Since the sets are dierently sized, they could reach the same sum in a variety of ways. If both sets are centered on zero, the sums could be zero; or, the smaller set could have a higher average, in which case the medians would be dierent. Statement (2) is also insu cient. If the median of Y is 0, Y = {-4, -3, -2, -1, 0, 1, 23, 4}, but we don t know anything about the median of X. Taken together, the statements are su cient. The sum of Y, as given in (2), is 0. If the sum of X is also 0, X = {-3, -2, -1, 0, 1, 2, 3}, which has a median of 0. Thus, the medians of the two sets are equal. Choice (C) is correct. 96. C Explanation: 3}. Statement (1) is insu cient. If the median is greater than -4, the median must be either 0 (if m is greater than 0) or m, if m is between -4 and 0. m could be anywhere from -4 up to a very large number. Statement (2) is also insu cient. If the median is -4, the median is negative, and m would be less than -4. If m is the median, as in (1), it could be a number between -4 and 0. Taken together, the statements are su cient. The only way the median is between -4 and 0 is if m is the median, in which case m is between -4 and 0, which is within the range of -6 and 2 that the question is asking about. Choice (C) is correct. 97. E Explanation: Statement (1) tells us that 20 have either only been immunized against D, or immunized against neither. Statement (2) tells us that 30 have either been immunized only against D, only against E, or against neither. Taken together, we can determine that 10 have been immunized only against E, but that leaves two variables. 20 is the sum of those who have been immunized only against D or against neither. Since there s no way of separating those groups, the correct choice is (E). 98. C Explanation: Statement (1) is insu cient: knowing the median doesn t, by itelf, tell you the mean. For the mean, you need to either know all the terms, or the relationships between all the terms. Statement (2) is also insu cient. Knowing the integers are equally spaced tells us that the median and the mean are equal, but since we don t know anything about the size of the numbers, we can t answer the question. Taken together, the statements are su cient. (2) tells us that the mean and the median are the same, while (1) gives us the median. Between them, we know the mean is 44. Choice (C) is correct. 99. C 50 Copyright 2008 Je Sackmann jesackmann@gmail.com - www.gmathacks.com

In order, the terms in the set, other than m, are {-9, -4, 0,

6. EXPLANATIONS

Je Sackmann GMAT Tutor jesackmann@gmail.com www.gmathacks.com

Explanation: Statement (1) is insu cient: all it tells us is that r is not the median. Statement (2) is also insu cient: if r = s, those two terms could equal the median. (In a set with an even number of terms, the only way a term equals the median is if the middle two terms are equal.) Taken together, we can answer the question. Since s, t, and u are consecutive, and r is smaller than all of them, all of the integers are dierent. Thus, the median is the average of two terms that are dierence, so the median is not equal to any of the terms in the set. Choice (C) is correct. 100. E Explanation: Statement (1) is insucient. A set of 5 consecutive even integers, each less than 30, could be as great as {20, 22, 24, 26, 28}, or as small as {2, 4, 6, 8, 10}. Not only do we not know the least integer in A, we know nothing about B. Statement (2) is also insu cient. B could be as great as {19, 21, 23} or as small as {1, 3, 5}. Again, we don t know the least integer, or anything about the other set. Taken together, the statements are still insu cient. The least integer in A could be anywhere from 2 to 20, and the least integer in B could be anywhere from 1 to 19. The two aren t dependent on each other in any way, so there s no way to know which is larger. Choice (E) is correct.

51 Copyright 2008 Je Sackmann jesackmann@gmail.com - www.gmathacks.com

También podría gustarte